Đến nội dung

Hình ảnh

Topic ôn luyện VMO 2015

vmo2015

  • Please log in to reply
Chủ đề này có 137 trả lời

#41
Bui Ba Anh

Bui Ba Anh

    Thiếu úy

  • Thành viên
  • 562 Bài viết

Bài 24 Cho $a;b;c$ là các số thực không âm sao cho $a+b+c>0$. CMR:

$\frac{a^2}{3a^2+(b+c)^2}+\frac{b^2}{3b^2+(a+c)^2}+\frac{c^2}{3c^2+(a+b)^2} \leq \frac{1}{2}$

A-L :)


NgọaLong

#42
mnguyen99

mnguyen99

    Thiếu úy

  • Thành viên
  • 696 Bài viết

Giải thử đi em, nói mà ko làm cụ thể ra là ko tốt đâu  :wub:

$n=2k+1\Rightarrow m=2^{3k+1}.a$

Theo đề ta có $\upsilon _{5}(10^{2^{3k+1}.a})=\upsilon _{5}(2^{6k+3}.a^{2}+2^{6k+3})\Leftrightarrow 2^{3k+1}.a=\upsilon _{5}(a^{2}+1)$

Đến lúc này ta chỉ cần áp dụng quy nạp để CM VT lớn hơn VP ở Th nào đó.

Bài toán đuộc CM xong.

PS:Anh Nam do em thấy đề này là của THTT chưa hết hạn nên em chỉ làm vậy thôi.


Bài viết đã được chỉnh sửa nội dung bởi mnguyen99: 12-11-2014 - 14:04

THCS NGUYỄN DUY,PHONG ĐIỀN$\Rightarrow$THPT CHUYÊN QUỐC HỌC HUẾ$\Rightarrow$??? 

 

TẬP LÀM THÁM TỬ TẠI ĐÂY http://diendantoanho...ám/#entry513026


#43
Hoang Tung 126

Hoang Tung 126

    Thiếu tá

  • Thành viên
  • 2061 Bài viết

Bài 24 Cho $a;b;c$ là các số thực không âm sao cho $a+b+c>0$. CMR:

$\frac{a^2}{3a^2+(b+c)^2}+\frac{b^2}{3b^2+(a+c)^2}+\frac{c^2}{3c^2+(a+b)^2} \leq \frac{1}{2}$

A-L :)

Bài 24 :

  Theo BĐT Bunhiacopxki có

 

 Ta có : $\sum \frac{4a^2}{3a^2+(b+c)^2}=\sum \frac{(a+a)^2}{(a^2+b^2+c^2)+(2a^2+bc)+bc}\leq \sum \frac{(a+a)^2}{(a^2+b^2+c^2)+(2a^2+bc)}\leq \sum (\frac{a^2}{a^2+b^2+c^2}+\frac{a^2}{2a^2+bc})=\frac{\sum a^2}{\sum a^2}+\sum \frac{a^2}{2a^2+bc}=1+\sum \frac{a^2}{2a^2+bc}$

 

    Do đó  ta cần Cm :

 

    $1+\sum \frac{a^2}{2a^2+bc}\leq 2< = > \sum \frac{a^2}{2a^2+bc}\leq 1< = > \sum (\frac{1}{2}-\frac{a^2}{2a^2+bc})\geq \frac{1}{2}< = > \sum \frac{bc}{2a^2+bc}\geq 1$

 

BĐT này đúng do $\sum \frac{bc}{2a^2+bc}=\sum \frac{b^2c^2}{2a^2bc+b^2c^2}\geq \frac{(\sum bc)^2}{2abc\sum a+\sum b^2c^2}=1$

 

   Do đó ta có ĐPCM. Dấu = xảy ra khi $a=0,b=c$ và các hoán vị tương ứng



#44
mnguyen99

mnguyen99

    Thiếu úy

  • Thành viên
  • 696 Bài viết

Trên Blog cũ của Toàn Zaraki có bài tương tự này anh  ^_^ 
Mặc dù còn 23 phút nữa mới đến ngày 12 nhưng em xin đăng một bài số thuộc đề KT đội tuyển Hải Dương mà em xin xỏ được

Bài 23: Tìm $(a,b,m,n)$ nguyên dương thỏa mãn: 
$$a^m-b^m=(a-b)^n$$

Chém luôn bài này.

ko mất tính tổng quát giả sử a>b            (a=b thì xong rồi)

Giả sử a-b tồn tại một ước nguyên tố lẻ thì

$\upsilon _{p}(a^{m}-b^{m})=n.\upsilon _{p}(a-b)\Rightarrow \upsilon _{p}(m)=(n-1)\upsilon _{p}(a-b)\Rightarrow m > p^{n-1}$

Do đó : $a^{p^{n-1}}-b^{p^{n-1}}< (a-b)^{n}$

Bằng pp quy nạp ta dễ dàng CM điều ngược lại.

Nên a-b ko tồn tại ước lẻ.

$a^{m}-b^{m}=2^{kn}\Rightarrow (a-b).\sum_{i=0}^{m-1}(a^{m-1-i}.b^{i})=2^{kn}\Rightarrow m.b^{m-1}\vdots 2$

+Nêu m là số chẵn, a,b lẻ thì $\upsilon _{2}(a^{m}-b^{m})=kn\Leftrightarrow \upsilon  _{2}(a^{2}-b^{2})+\upsilon _{2}(m)-1=kn\Leftrightarrow k+\upsilon _{2}(a+b)+\upsilon _{2}(m)-1=kn\Rightarrow \upsilon _{2}(2b+2^{k})+k+\upsilon _{2}(m)-1=kn\Leftrightarrow k+\upsilon _{2}(m)=kn\Rightarrow \upsilon _{2}(m)=k(n-1)$

áp dụng phương pháp trên ta có điều vô lí.

+Nếu a,b là số chẵn

Ta chỉ xét th n lớn tức 

   -Giả sử a-b=2$\Rightarrow (2+b)^{m}+b^{m}=2^{n}$

   Trong hai số 2+b và b tồn tại một số ko chia hết cho 4 và một số chia hết cho 4.                                         

     LÀm tiếp \\\

    -0Xét a-b chia hết cho 4.

      $\upsilon _{2}(a^{m}-b^{m})=kn\Leftrightarrow \upsilon _{2}(a-b)+\upsilon _{2}(m)=kn\Leftrightarrow\upsilon _{2}(m)=k(n-1)$

 Từ đó ta tìm ra kq bài toán


THCS NGUYỄN DUY,PHONG ĐIỀN$\Rightarrow$THPT CHUYÊN QUỐC HỌC HUẾ$\Rightarrow$??? 

 

TẬP LÀM THÁM TỬ TẠI ĐÂY http://diendantoanho...ám/#entry513026


#45
Juliel

Juliel

    Thượng úy

  • Thành viên
  • 1240 Bài viết

Chém luôn bài này.

ko mất tính tổng quát giả sử a>b            (a=b thì xong rồi)

Giả sử a-b tồn tại một ước nguyên tố lẻ thì

$\upsilon _{p}(a^{m}-b^{m})=n.\upsilon _{p}(a-b)\Rightarrow \upsilon _{p}(m)=(n-1)\upsilon _{p}(a-b)\Rightarrow m > p^{n-1}$

Do đó : $a^{p^{n-1}}-b^{p^{n-1}}< (a-b)^{n}$

Bằng pp quy nạp ta dễ dàng CM điều ngược lại.

Nên a-b ko tồn tại ước lẻ.

$a^{m}-b^{m}=2^{kn}\Rightarrow (a-b).\sum_{i=0}^{m-1}(a^{m-1-i}.b^{i})=2^{kn}\Rightarrow m.b^{m-1}\vdots 2$

+Nêu m là số chẵn, a,b lẻ thì $\upsilon _{2}(a^{m}-b^{m})=kn\Leftrightarrow \upsilon  _{2}(a^{2}-b^{2})+\upsilon _{2}(m)-1=kn\Leftrightarrow k+\upsilon _{2}(a+b)+\upsilon _{2}(m)-1=kn\Rightarrow \upsilon _{2}(2b+2^{k})+k+\upsilon _{2}(m)-1=kn\Leftrightarrow k+\upsilon _{2}(m)=kn\Rightarrow \upsilon _{2}(m)=k(n-1)$

áp dụng phương pháp trên ta có điều vô lí.

+Nếu a,b là số chẵn

Ta chỉ xét th n lớn tức 

   -Giả sử a-b=2$\Rightarrow (2+b)^{m}+b^{m}=2^{n}$

   Trong hai số 2+b và b tồn tại một số ko chia hết cho 4 và một số chia hết cho 4.                                         

     LÀm tiếp \\\

    -0Xét a-b chia hết cho 4.

      $\upsilon _{2}(a^{m}-b^{m})=kn\Leftrightarrow \upsilon _{2}(a-b)+\upsilon _{2}(m)=kn\Leftrightarrow\upsilon _{2}(m)=k(n-1)$

 Từ đó ta tìm ra kq bài toán

Em dùng LTE cho chỗ này thì phải đảm bảo là $p\nmid a$ và $p\nmid b$.

Em phải làm rõ ra 1 số chỗ chứ đừng viết qua loa để người đọc phải tìm lời giải chứ >.<

 

Anh cũng ko hiểu bài làm $10^m-8^n=2m^2$ của em, nhưng vì nó trên TTT nên thôi ko bàn luận nữa ~


Bài viết đã được chỉnh sửa nội dung bởi Juliel: 12-11-2014 - 19:00

Đừng rời xa tôi vì tôi lỡ yêu người mất rồi !
 

Welcome to My Facebook !


#46
Zaraki

Zaraki

    PQT

  • Phó Quản lý Toán Cao cấp
  • 4273 Bài viết

Bài 25. Giải phương trình nghiệm nguyên $406+3^n+9^n+60^n=m^4.$

Bài này mình nghĩ mãi không ra mặc dù có đoán được nghiệm.  :(


Discovery is a child’s privilege. I mean the small child, the child who is not afraid to be wrong, to look silly, to not be serious, and to act differently from everyone else. He is also not afraid that the things he is interested in are in bad taste or turn out to be different from his expectations, from what they should be, or rather he is not afraid of what they actually are. He ignores the silent and flawless consensus that is part of the air we breathe – the consensus of all the people who are, or are reputed to be, reasonable.

 

Grothendieck, Récoltes et Semailles (“Crops and Seeds”). 


#47
WhjteShadow

WhjteShadow

    Thượng úy

  • Phó Quản lý Toán Ứng dụ
  • 1323 Bài viết

 

Bài 13: Cho $35$ người đi dự một buổi họp. Có tất cả $112$ căp hai người quen nhau. Chứng minh rằng tồn tại $4$ người $a,b,c,d$ sao cho $a$ quen $b$, $b$ quen $c$, $c$ quen $d$ và $d$ quen $a$.

Ta viết lại bài toán dưới dạng ngôn ngữ đồ thị :

Có $35$ điểm trên mặt phẳng, và có tổng cộng $112$ đoạn thẳng nối các điểm này. Chứng minh rằng tồn tại 1 chu trình 4 cạnh (đường gấp khúc 4 đỉnh 4 cạnh)

Giả sử phản chứng, không tồn tại 1 chu trình 4 cạnh nào. Ta sẽ đếm số góc của đồ thị này theo 2 cách rồi so sánh :

Cách 1 : Với mỗi cặp điểm $A, B$ trên mặt phẳng, tồn tại nhiều nhất 1 góc với điểm ở đỉnh góc khác 2 điểm thuộc cặp điểm vừa nói. Thật vậy vì nếu có 2 góc như vậy (giả sử điểm ở đỉnh lần lượt là $C,D$) thì 

Untitled.png  

ta sẽ có chu trình $ACBDA$ tương đương với $A$ quen $C$, $C$ quen $B$, $B$ quen $D$, $D$ quen $A$ (trái với điều giả sử). Mà có $35$ đỉnh

Vậy nên sẽ có nhiều nhất là $C^2_{35}$ góc.

Cách 2 : Giả sử điểm $i$ được nối với $d_i$ điểm khác ($i=\overline{1;35}$) $\Rightarrow$ sẽ có $d_i$ cạnh nhận điểm $i$ là đỉnh. Vậy số góc có đỉnh là $i$ là $C^2_{d_i}$.

Dễ dàng chứng minh được $\sum^{35}_{i=1} d_i = 2.112=224$ (Bằng cách để ý 1 cạnh được nối vào đúng 2 điểm).

Vậy số góc là :

$$\sum^{35}_{i=1} $C^2_{d_i}=\frac{\sum^{35}_{i=1}d_i^2-\sum^{35}_{i=1}d_i}{2}\\ \geq_{Cauchy-Schwarz} \dfrac{\frac{(\sum^{35}_{i=1}d_i)^2}{35}-224}{2}=\frac{224^2/35-224}{2}$$

Từ 2 cách đếm, ta có $595=C^2_{35}\geq \frac{224^2/35-224}{2}>604$ (Vô lí).

Vậy giả sử ban đầu là sai, ta có dpcm !


“There is no way home, home is the way.” - Thich Nhat Hanh

#48
WhjteShadow

WhjteShadow

    Thượng úy

  • Phó Quản lý Toán Ứng dụ
  • 1323 Bài viết

Bài 22:

 

Cho tam giác $ABC$ và 1 đường tròn $(O)$. Gọi $A'B'C'$ là tam giác đường tạo thành bởi 3 đường đối cực của lần lượt $A, B, C$ với $(O)$.

Khi đó $AA', BB', CC'$ đồng quy.

Untitled.png

Vẽ $D,E,F$ lần lượt là hình chiếu của $O$ lên $BC,CA,AB$, $H$ là trực tâm tam giác $A',B',C'$. $A'H,B'H,C'H$ lần lượt cắt $B'C',A'C',A'B'$ tại $M,N,P$. $B'C',C'A',A'B'$ lần lượt cắt $BC,CA,AB$ tại $X,Y,Z$. 

Do $(MX;MA')\equiv (DX;DA')\equiv \frac{\pi}{2}\pmod{\pi}$ nên $XMDA'$ nội tiếp đường tròn đường kính $A'X$.

Tương tự ta có $YNEB'$ nội tiếp đường tròn đường kính $B'Y$, $ZPFC'$ nội tiếp đường tròn đường kính $C'Z$.

Mà $\overline{HM}.\overline{HA'}=\overline{HN}.\overline{HB'}=\overline{HP}.\overline{HC'}$ (Do $H$ là trực tâm $A'B'C'$), $\overline{OD}.\overline{OA'}= \overline{OE}.\overline{OB'}=\overline{OF}.\overline{OC'}$ (Từ giả thiết).

$\Rightarrow \, H,O$ có cùng phương tích với 3 đường tròn đường kính $A'X,B'Y,C'Z$. 

$\Rightarrow$ tâm 3 đường tròn đường kính $A'X,B'Y,C'Z$ thẳng hàng hay trung điểm $A'X,B'Y,C'Z$ thẳng hàng.

Vẽ $A'C'$ cắt $XZ$ tại $Y'$ suy ra trung điểm $XA',ZC',Y'B'$ thẳng hàng (đường thẳng Gauss với tứ giác toàn phần $XC'A'ZY'B'$)

Vậy nên trung điểm trung điểm $XA',ZC',Y'B',YB'$ thẳng hàng trên đường thẳng song song $YY'$ (Đường trung bình) từ đó suy ra $Y$ trùng $Y'$.

Vậy $X',Y',Z'$ thẳng hàng. Áp dụng định lý $Desargue$ cho tam giác $ABC$ và $A'B'C'$ có ngay đpcm !

------------------------

Đôi lời muốn nói với mọi người trong topic :

Minh nghĩ là chúng ta nên hạn chế post những bài toán số học thuần túy, thay vào đó là thêm những bài phương trình, hệ phương trình, dãy số. Lý do thì ai cũng biết, topic ôn thi cái gì thì chỉ nên post nhưng thứ mà kì thi đó ra thôi đúng không ? Và còn 1 điều nữa là mong các anh/chị/bạn không post những bài khó tầm cỡ TST hay cao hơn. Và điều cuối mình thiết nghĩ nên có thêm 1 luật là bài nào 1 tuần rồi chưa có ai giải được thì người post nên chia sẻ lời giải cho mọi người tham khảo cùng, vậy sẽ tạo nên môi trường ôn luyện trao đổi học tập tốt hơn.


“There is no way home, home is the way.” - Thich Nhat Hanh

#49
namcpnh

namcpnh

    Red Devil

  • Hiệp sỹ
  • 1153 Bài viết

 

Đôi lời muốn nói với mọi người trong topic :

Minh nghĩ là chúng ta nên hạn chế post những bài toán số học thuần túy, thay vào đó là thêm những bài phương trình, hệ phương trình, dãy số. Lý do thì ai cũng biết, topic ôn thi cái gì thì chỉ nên post nhưng thứ mà kì thi đó ra thôi đúng không ? Và còn 1 điều nữa là mong các anh/chị/bạn không post những bài khó tầm cỡ TST hay cao hơn. Và điều cuối mình thiết nghĩ nên có thêm 1 luật là bài nào 1 tuần rồi chưa có ai giải được thì người post nên chia sẻ lời giải cho mọi người tham khảo cùng, vậy sẽ tạo nên môi trường ôn luyện trao đổi học tập tốt hơn.

 

Anh ủng hộ ý kiến của em, tuy nhiên việc cấm các mem post các bài tầm TST trở lên cũng khó vì đôi khi trong các kì thi QG người ra đề vẫn lấy ý tưởng của đề TST các năm cũ làm đề năm đó luôn.

Tuy nhiên ta có thể thêm 1 đk ràng buộc là ai post bài thì người đó phải chịu trách nhiệm về đáp án bài đó.


Cùng chung sức làm chuyên đề hay cho diễn đàn tại :

Dãy số-giới hạn, Đa thức , Hình học , Phương trình hàm , PT-HPT-BPT , Số học.

Wolframalpha đây


#50
BlackSelena

BlackSelena

    $\mathbb{Sayonara}$

  • Hiệp sỹ
  • 1549 Bài viết

Bài 26:

Cho tam giác $ABC$, $M$ là 1 điểm trong tam giác. $AM, BM, CM$ cắt $BC, CA, AB$ tại $A_1, B_1, C_1$. $(AB_1C_1)$ cắt $(O)$ tại $A_2$. Tuơng tự có $B_2, C_2$. $X \equiv B_2C_2 \cap BC$, tương tự có $Y, Z$.

Chứng minh $O$ là trực tâm $\triangle XYZ$



#51
Hoang Tung 126

Hoang Tung 126

    Thiếu tá

  • Thành viên
  • 2061 Bài viết

 Nghe góp ý của anh WhjteShadow, em xin góp 1 bài dãy

 

    Bài 27 :Tìm CTTQ của $x_{n}$ thỏa mãn $\left\{\begin{matrix} x_{1}=\frac{3}{\sqrt{6}} & \\ x_{n+1}=24x_{n}^3-12\sqrt{6}x_{n}^2+15x_{n}-\sqrt{6} & \end{matrix}\right.$  với mọi số tự nhiên $n\geq 1$



#52
Juliel

Juliel

    Thượng úy

  • Thành viên
  • 1240 Bài viết

 Nghe góp ý của anh WhjteShadow, em xin góp 1 bài dãy

 

    Bài 27 :Tìm CTTQ của $x_{n}$ thỏa mãn $\left\{\begin{matrix} x_{1}=\frac{3}{\sqrt{6}} & \\ x_{n+1}=24x_{n}^3-12\sqrt{6}x_{n}^2+15x_{n}-\sqrt{6} & \end{matrix}\right.$  với mọi số tự nhiên $n\geq 1$

Lời giải :

Ta đặt $u_n=x_n\sqrt{6}-1$ ta được dãy $(u_n)$ :

$$\left\{\begin{matrix} u_1=2\\ u_{n+1}=4u_n^3-3u_n \end{matrix}\right.$$

Xét số thực $a>1$ để :

$$2=u_1=\dfrac{1}{2}\left ( a+\dfrac{1}{a} \right )\Leftrightarrow a=2+\sqrt{3}$$

Khi đó :

$$u_2=4.\left [ \dfrac{1}{2}\left ( a+\dfrac{1}{a} \right ) \right ]^3-3.\left [ \dfrac{1}{2}\left ( a+\dfrac{1}{a} \right ) \right ]=\dfrac{1}{2}\left ( a^{3^1}+\dfrac{1}{a^{3^1}} \right )$$

Quy nạp được :

$$u_{n}=\dfrac{1}{2}\left ( a^{3^{n-1}}+\dfrac{1}{a^{3^{n-1}}} \right )$$

Từ đó đuơc :

$$x_n=\frac{1}{2\sqrt{6}}\left ( \left ( 2+\sqrt{3} \right )^{3^{n-1}}+\left ( 2-\sqrt{3} \right )^{3^{n-1}}+2 \right )$$

 

Bài 28 : Cho hai dãy số dương $(x_n),(y_n)$ xác định bởi $x_1=y_1=\dfrac{3}{\sqrt{2}}$ và :

$$\left\{\begin{matrix} 9x_{n+1}=4x_{n+1}y_{n+1}^2-9x_n\\ 9y_{n+1}=4y_{n+1}x^2_{n+1}+9y_n \end{matrix}\right.,\;\forall n\in \mathbb{N}^*$$

Chứng minh hai dãy này có giới hạn hữu hạn và tính các giới hạn này.


Đừng rời xa tôi vì tôi lỡ yêu người mất rồi !
 

Welcome to My Facebook !


#53
banhgaongonngon

banhgaongonngon

    Thượng úy

  • Thành viên
  • 1046 Bài viết

Bài 28 : Cho hai dãy số dương $(x_n),(y_n)$ xác định bởi $x_1=y_1=\dfrac{3}{\sqrt{2}}$ và :

$$\left\{\begin{matrix} 9x_{n+1}=4x_{n+1}y_{n+1}^2-9x_n\\ 9y_{n+1}=4y_{n+1}x^2_{n+1}+9y_n \end{matrix}\right.,\;\forall n\in \mathbb{N}^*$$

Chứng minh hai dãy này có giới hạn hữu hạn và tính các giới hạn này.

 

Ta chứng minh quy nạp theo $n$ rằng $x_{n}^{2}+y_{n}^{2}=9$     $(*)$

 

* Với $n=1$, hiển nhiên $(*)$ đúng

 

* Giả sử $(*)$ đúng với $n$, tức là $x_{n}^{2}+y_{n}^{2}=9$

 

Ta có

$(9x_{n})^{2}+(9y_{n})^{2}=\left ( 9x_{n+1}-4x_{n+1}y_{n+1}^{2} \right )^{2}+\left ( 9y_{n+1}-4y_{n+1}x_{n+1}^{2} \right )^{2}$

 

$\Rightarrow \left ( x _{n+1}^{2}+y_{n+1}^{2}-9\right )\left ( 16x_{n+1}^{2}y_{n+1}^{2} +81\right )=0$

 

Điều này khẳng định $(*)$ đúng với $n+1$, theo nguyên lí quy nạp $(*)$ đúng với mọi số nguyên dương $n$

 

Từ $(*)$ ta thấy với mỗi $n$ thì $-3\leq x_{n},y_{n}\leq 3$

 

Cũng bằng quy nạp ta có được

$\left\{\begin{matrix} x_{n}=3 \sin \frac{\pi }{4.3^{n-1}}\\ y_{n}=3 \cos \frac{\pi }{4.3^{n-1}} \end{matrix}\right.$

 

Kết luận:

$\boxed {\lim x_{n}=0, \lim y_{n}=3}$

 

_______________________________________________________________________________

 

Xin góp thêm cho topic một bài tự chế được :D

 

Bài 29   Cho dãy số $(x_{n})$ xác định bởi $\left\{\begin{matrix} x_{1}=a > 0\\ x_{n+1}= \ln \left (1+ x_{n} \right ),\forall n\in \mathbb{N} \end{matrix}\right.$

 

1. Chứng minh rằng với mọi số thực $M > 1$ luôn tồn tại số nguyên dương $k$ thỏa mãn $1\leq x_{n+1}+\frac{1}{x_{n}+1}\leq M,\forall n\geq k$

 

2. Tìm tất cả các số thực $s$ sao cho $n^{s}\left ( x_{n}-x_{n+1} \right )$ có giới hạn hữu hạn khác $0$


Bài viết đã được chỉnh sửa nội dung bởi namcpnh: 16-11-2014 - 16:31


#54
WhjteShadow

WhjteShadow

    Thượng úy

  • Phó Quản lý Toán Ứng dụ
  • 1323 Bài viết

Bài 10:(Đề thi trại hè hùng vương 2014)

Chứng minh rằng:tồn tại $16$ số tự nhiên liên tiếp sao cho không có số nào trong $16$ số có thể biểu diễn dưới dạng $\left | 7x^2+9xy-5y^2 \right |$($a,b\in R$)

Mọi người xem tại đây

Đầu tiên ta tìm hiểu tính chất của ước nguyên tố của $7x^2+9xy-5y^2$. 

Gọi $p$ là 1 ước nguyên tố của $7x^2+9xy-5y^2$ ($x,y\in \mathbb{R}$). 

Nếu 1 trong 2 số $x,y\vdots p$ thì $x,y$ đều $\vdots p$ từ đó $p^2| 7x^2+9xy-5y^2$. 

Nếu $(x;p)=(y;p)=1$ thì $p|7x^2+9xy-5y^2\Rightarrow p|140x^2+180xy-100y^2=221x^2-(9x-10y)^2$

$\Rightarrow 221x^2\equiv (9x-10y)^2 \pmod{p}$ mà $(x;p)=1\Rightarrow $ 221 là số chính phương mod $p$ hay $\left(\frac{221}{p}\right)=1$ (Kí hiệu Jacobi)

Đế ý $221=13.17$, $13,17$ là 2 số nguyên tố dạng $4k+1$, vậy nên theo tính chất của kí hiệu Jacobi và luật tương hỗ Gauss thì :

$$1=\left(\frac{221}{p}\right)=\left(\frac{13}{p}\right).\left(\frac{17}{p}\right)=\left(\frac{p}{13}\right).\left(\frac{p}{17}\right)$$

Vậy nên nếu $(x;p)=(y;p)=1$ thì $7x^2+9xy-5y^2$ chỉ có các ước nguyên tố $p$ mà $\left(\frac{p}{13}\right).\left(\frac{p}{17}\right)=1$

Từ đây ta thấy để chỉ ra 1 số không có dạng $| 7x^2+9xy-5y^2|$ thì ta sẽ chỉ ra nó có ước nguyên tố $p$ là scp mod 13 mà không là scp mod 17 hoặc ngược lại.

Dễ thấy $p$ là scp mod 13 $\Leftrightarrow$ $p$ chia $13$ dư $1;3;4;9;10;12$, $p$ là scp mod 17 $\Leftrightarrow$ $p$ chia $17$ dư $1,2,4,8,9,13,15,16$

Vậy nên ta dễ dàng tính được 16 số nguyên tố là scp mod 13 mà không là scp mod 17 hoặc ngược lại là $\{p_1;p_2;...;p_{16}\}=\{2;3;13;17;19;23;29;47;59;61;67;79;83;89;107;113\}$ (Phù !)

Vậy theo định lí thặng dư Trung Hoa, tồn tại $n$ nguyên dương để hệ $n\equiv -i+p_i\pmod{p_i^2}$ ($i=\overline{1;16}$) có nghiệm. Lúc đó xét 16 số nguyên dương liên tiếp $n+1,n+2,...n+16$, có $n+i\vdots p_i$ mà $\not \vdots p_i^2$ nên giả sử phản chứng tồn tại $x,y$ nguyên : $n+i=|7x^2+9xy-5y^2|$

Lúc đó $(x;p_i)=(y;p_i)=1$ (Do nếu $x\vdots p_i$ thì $n=|7x^2+9xy-5y^2|\vdots p_i^2$ mâu thuẫn) mà $p_i$ nguyên tố, $\left(\frac{p_i}{13}\right).\left(\frac{p_i}{17}\right)=-1$, $p_i |n+i =|7x^2+9xy-5y^2| $ (mâu thuẫn tiếp).

Vậy $n+i$ không có dạng $|7x^2+9xy-5y^2|$ với $i=\overline{1;16}$. Ta có điều phải chứng minh !


Bài viết đã được chỉnh sửa nội dung bởi namcpnh: 09-12-2014 - 23:53

“There is no way home, home is the way.” - Thich Nhat Hanh

#55
Juliel

Juliel

    Thượng úy

  • Thành viên
  • 1240 Bài viết

Đây là lời giải bài 10 mà em tìm được từ đáp án chính thức của kỳ thi nhưng thấy nó ko tự nhiên bằng lời giải của a WhiteShadow, bá vãi ~~

THHV.JPG


Đừng rời xa tôi vì tôi lỡ yêu người mất rồi !
 

Welcome to My Facebook !


#56
namcpnh

namcpnh

    Red Devil

  • Hiệp sỹ
  • 1153 Bài viết

Bài 30: 

Hai người An và Bình chơi trò chơi bốc kẹo. Trên bàn có $2015$ viên kẹo . Số kẹo mà hai người có thể bốc là ${1;2;6}$.

a) Hỏi ai là người có chiến lược thắng?

b) Câu hỏi tương tự với 2016 viên kẹo? 


Bài viết đã được chỉnh sửa nội dung bởi LNH: 17-11-2014 - 20:53

Cùng chung sức làm chuyên đề hay cho diễn đàn tại :

Dãy số-giới hạn, Đa thức , Hình học , Phương trình hàm , PT-HPT-BPT , Số học.

Wolframalpha đây


#57
Ispectorgadget

Ispectorgadget

    Nothing

  • Quản lý Toán Phổ thông
  • 2946 Bài viết

Bài 31: Cho $a_1,a_2,a_3...$ là một dãy số thực dương. Giả sử với một số nguyên dương $s$ cho trước, ta có $$a_n=\max\{a_k.a_{n-k}:1\le k\le n-1\}$$

với mọi $n>s$. Chứng minh rằng tồn tại các số nguyên dương $l$ và $N$, với $l\le s$ thỏa mãn $a_n=a_l.a_{n-l} \forall n\ge N$.

Bài viết đã được chỉnh sửa nội dung bởi WhjteShadow: 19-11-2014 - 16:04

►|| The aim of life is self-development. To realize one's nature perfectly - that is what each of us is here for. ™ ♫


#58
namcpnh

namcpnh

    Red Devil

  • Hiệp sỹ
  • 1153 Bài viết

Chú ý những người khác không nên ra đề nữa, hãy tập trung vào những bài mình tổng hợp và đăng thêm

 

Bài 32: 

Cho trước số nguyên dương n. Xét đa thức P(x) và Q(x) không là đa thức 0 có hệ số thực thỏa mãn: 

i) Bậc của P(x) nhỏ hơn n+1

ii) .Q(x) là đa thức chuẩn tắc bậc n +1 có đúng n+1 nghiệm phân biệt b_i đã sắp thứ tự.

iii) P(b_i).Q'(b_i) <0 với mọi số b_i.

 Chứng minh rằng P(x) không có nghiệm bội

 

Bài 33: Cho dãy số xác định bởi:

$\left\{\begin{matrix} u_0=1, u_1=-1\\ u_{n+1}=ku_n-u_{n-1},\forall n \in \mathbb{N} \end{matrix}\right.$

Tìm tất cả các giả trị $k \in \mathbb{Q}$ để  $(u_n)$ là dãy tuần hoàn.


Bài viết đã được chỉnh sửa nội dung bởi WhjteShadow: 19-11-2014 - 17:46

Cùng chung sức làm chuyên đề hay cho diễn đàn tại :

Dãy số-giới hạn, Đa thức , Hình học , Phương trình hàm , PT-HPT-BPT , Số học.

Wolframalpha đây


#59
Karl Heinrich Marx

Karl Heinrich Marx

    Sĩ quan

  • Thành viên
  • 321 Bài viết

Bài 30:

Ta định nghĩa một vị trí thắng (W) nếu bắt đầu từ vị trí đó người chơi thắng và thua(L) nếu bắt đầu từ vị trí đó người chơi sẽ thua.

Ta có thể phát biểu những vị trí này theo một cách khác:

Một vị trí là W nếu tồn tại ít nhất một cách đi từ vị trí đó đến một vị trí L

Một vị trí là L nếu với mọi cách đi ta đều đi vào vị trí W.

Định nghĩa trên đưa ra thực chất để ta kiểm tra một cách nhanh chóng kết quả với những số nhỏ ban đầu để suy ra tính chất bài toán, sau khi biết quy luật ta có thể quy nạp hoặc tìm cách chứng minh mau chóng hơn.

Mình sẽ thử làm với bài này từ cách xác định trên

Ta giả sử người không thể bốc nữa là thua thì

ta có 0L,1W,2W,3L,4W,5W,6W,7L,8W,9W,10L,11W,12W,...

cứ mỗi số $n$ ta đem nó trừ cho 1,2,6 nếu đều rơi vào số W thì nó là L ngược lại nếu ít nhất một lần rơi vào số L thì nó là W. Theo mình thử thì có vẻ như dãy này tuần hoàn lại bắt đầu từ số 18 (mình ko chắc lắm các bạn có thể kiểm tra rõ hơn). Có thể chứng minh quy nạp các số $18n,18n+3,18n+7,18n+10,...$ là vị trí L còn các vị trí còn lại là W. Từ đây có thể kiểm tra ngay 2 số ở đầu bài là W hay L.

Nếu đề bài là người thắng là người bốc được viên cuối cùng thì tất các vị trí trên đổi lại từ L thành W và từ W thành L.

Phương pháp này có vẻ thủ công và nguyên lí đơn giản nhưng có thể dùng để giải rất nhiều các bài toán trò chơi.



#60
hoangtubatu955

hoangtubatu955

    Sĩ quan

  • Thành viên
  • 429 Bài viết

Bài 30. Ngoài cách trên thì em có một cách như sau:
 

Đề em chưa biết là bắt viên cuối thắng hay thua, thôi thì giả sử bắt viên cuối cùng thắng.

Xét theo mod 7 thì ta có cấu hình thì sẽ có những cấu hình xét theo mod 7 như sau:
Cấu hình thắng: $1,2,4,5,6$
Cấu hình thua $3,7$
Do trường hợp $2016$ chia hết cho 7 nên hiển nhiên rằng người thứ 2 là thắng!
Bây giờ thì xét đến đến $2015$, do đây là một số thuộc nhóm đồng dư $6 mod 7$ nên người một sẽ luôn có chiến thuật thắng.
Như vậy, khi đó bài toán chứng minh xong!


Bài viết đã được chỉnh sửa nội dung bởi hoangtubatu955: 17-11-2014 - 18:56






Được gắn nhãn với một hoặc nhiều trong số những từ khóa sau: vmo2015

0 người đang xem chủ đề

0 thành viên, 0 khách, 0 thành viên ẩn danh